Second order accurate numerical approximation for first derivative Announcing the arrival of Valued Associate #679: Cesar Manara Planned maintenance scheduled April 23, 2019 at 23:30 UTC (7:30pm US/Eastern)Taylor expansion: first derivative approximation with third orderBoundary Conditions for a Finite Difference Approximation of a Sixth Derivative“Symmetric” numerical computation of second derivativeNumerical differentiation (approximation with three supporting points )Second order one-sided finite difference approximation to a partial derivative4th order accurate difference formula less accurate than 2nd order formula?Approximation of the first derivative by writing Taylor expansionsDerivation of fourth-order accurate formula for the second derivativeApproximation of $log(x)$ for very small $x$Second order approximation of first derivative gives odd results

What is a more techy Technical Writer job title that isn't cutesy or confusing?

Did Mueller's report provide an evidentiary basis for the claim of Russian govt election interference via social media?

Trying to understand entropy as a novice in thermodynamics

I can't produce songs

Was Kant an Intuitionist about mathematical objects?

Special flights

Is multiple magic items in one inherently imbalanced?

How can I prevent/balance waiting and turtling as a response to cooldown mechanics

One-one communication

Is there public access to the Meteor Crater in Arizona?

Why is the change of basis formula counter-intuitive? [See details]

Is it dangerous to install hacking tools on my private linux machine?

Why BitLocker does not use RSA

AppleTVs create a chatty alternate WiFi network

How does light 'choose' between wave and particle behaviour?

I got rid of Mac OSX and replaced it with linux but now I can't change it back to OSX or windows

Should a wizard buy fine inks every time he want to copy spells into his spellbook?

Why complex landing gears are used instead of simple,reliability and light weight muscle wire or shape memory alloys?

What is the "studentd" process?

The Nth Gryphon Number

What does the writing on Poe's helmet say?

Sally's older brother

Can an iPhone 7 be made to function as a NFC Tag?

How do living politicians protect their readily obtainable signatures from misuse?



Second order accurate numerical approximation for first derivative



Announcing the arrival of Valued Associate #679: Cesar Manara
Planned maintenance scheduled April 23, 2019 at 23:30 UTC (7:30pm US/Eastern)Taylor expansion: first derivative approximation with third orderBoundary Conditions for a Finite Difference Approximation of a Sixth Derivative“Symmetric” numerical computation of second derivativeNumerical differentiation (approximation with three supporting points )Second order one-sided finite difference approximation to a partial derivative4th order accurate difference formula less accurate than 2nd order formula?Approximation of the first derivative by writing Taylor expansionsDerivation of fourth-order accurate formula for the second derivativeApproximation of $log(x)$ for very small $x$Second order approximation of first derivative gives odd results










0












$begingroup$


We just learnt about one sided and centred difference approximations in class and we have been given a problem to find $a_0, a_1$ and $a_2$ in the below numerical approximation for a first derivative in order to make the approximation second order accurate:$$F'(x) approx frac1Delta x[a_0F(x + Delta x) + a_1F(x + 2 Delta x) + a_2F(x + 3 Delta x)].$$
Would it be right to say that we want the RHS to be in this form: $$fracF(x + Delta x) - F(x -Delta x)2 Delta x$$ and so, somehow, we'd manipulate the Taylor expansion of each $F(...)$ term in the first derivative approximation equation?










share|cite|improve this question









$endgroup$
















    0












    $begingroup$


    We just learnt about one sided and centred difference approximations in class and we have been given a problem to find $a_0, a_1$ and $a_2$ in the below numerical approximation for a first derivative in order to make the approximation second order accurate:$$F'(x) approx frac1Delta x[a_0F(x + Delta x) + a_1F(x + 2 Delta x) + a_2F(x + 3 Delta x)].$$
    Would it be right to say that we want the RHS to be in this form: $$fracF(x + Delta x) - F(x -Delta x)2 Delta x$$ and so, somehow, we'd manipulate the Taylor expansion of each $F(...)$ term in the first derivative approximation equation?










    share|cite|improve this question









    $endgroup$














      0












      0








      0





      $begingroup$


      We just learnt about one sided and centred difference approximations in class and we have been given a problem to find $a_0, a_1$ and $a_2$ in the below numerical approximation for a first derivative in order to make the approximation second order accurate:$$F'(x) approx frac1Delta x[a_0F(x + Delta x) + a_1F(x + 2 Delta x) + a_2F(x + 3 Delta x)].$$
      Would it be right to say that we want the RHS to be in this form: $$fracF(x + Delta x) - F(x -Delta x)2 Delta x$$ and so, somehow, we'd manipulate the Taylor expansion of each $F(...)$ term in the first derivative approximation equation?










      share|cite|improve this question









      $endgroup$




      We just learnt about one sided and centred difference approximations in class and we have been given a problem to find $a_0, a_1$ and $a_2$ in the below numerical approximation for a first derivative in order to make the approximation second order accurate:$$F'(x) approx frac1Delta x[a_0F(x + Delta x) + a_1F(x + 2 Delta x) + a_2F(x + 3 Delta x)].$$
      Would it be right to say that we want the RHS to be in this form: $$fracF(x + Delta x) - F(x -Delta x)2 Delta x$$ and so, somehow, we'd manipulate the Taylor expansion of each $F(...)$ term in the first derivative approximation equation?







      derivatives numerical-methods






      share|cite|improve this question













      share|cite|improve this question











      share|cite|improve this question




      share|cite|improve this question










      asked Apr 2 at 10:56









      FrancisFrancis

      393




      393




















          2 Answers
          2






          active

          oldest

          votes


















          2












          $begingroup$

          The formula you mention is a centered difference, but you are required to obtain a forward difference. You just need to use Taylor's formula and compute the coefficients that cancel out the lower order terms.



          $$
          F(x+Delta x)= F(x)+F'(x) Delta x + fracF''(x)2 (Delta x)^2 + O((Delta x)^3)
          $$



          $$
          F(x+ 2 Delta x)= F(x)+F'(x) 2 Delta x + fracF''(x)2 (2 Delta x)^2 + O((Delta x)^3)
          $$



          $$
          F(x+3Delta x)= F(x)+F'(x) 3Delta x + fracF''(x)2 (3Delta x)^2 + O((Delta x)^3)
          $$



          so you see that
          beginalign*
          F'(x)-frac1Delta x & (a_0 F(x+Delta x)+a_1 F(x+2Delta x) + a_2 F(x+3 Delta x))\
          =& frac1Delta xleft(F'(x) Delta x - a_0 (F(x)+F'(x) Delta x + fracF''(x)2 (Delta x)^2 + O((Delta x)^3))right.\
          & - a_1 (F(x)+F'(x) 2 Delta x + fracF''(x)2 (2 Delta x)^2 + O((Delta x)^3))\
          & left.- a_2 (F(x)+F'(x) 3Delta x + fracF''(x)2 (3Delta x)^2 + O((Delta x)^3))right)\
          =& frac1Delta xleft( -(a_0+a_1+a_2)F(x)+(1-a_0-2a_1-3a_2)F'(x)Delta xright.\
          & left.-(a_0+4a_1+9a_2)F''(x) (Delta x)^2/2 + O((Delta x)^3)right)
          endalign*



          To get the desired accuracy you just have to set $a_0, a_1, a_2$ such that



          $$
          a_0+a_1+a_2=0, quad a_0+2a_1+3a_2 = 1, quad a_0+4a_2+9a_2=0,
          $$



          that is
          $$
          a_0 = -frac 52, quad a_1 =4, quad a_2=-frac 32
          $$



          and the formula becomes
          $$
          F'(x) approx frac12Delta x left(-5 F(x+Delta x)+8F(x+2 Delta x) - 4F(x+3 Delta x) right)
          $$






          share|cite|improve this answer









          $endgroup$




















            1












            $begingroup$

            $a_0f(x+Delta x) = a_0f(x) + a_0Delta xf'(x) + a_0frac12Delta x^2f'(x) + a_0frac16Delta x^3f'''(xi_1)$



            $a_1f(x+2Delta x) = a_1f(x) + a_12Delta xf'(x) +
            a_12Delta x^2f''(x) + a_1frac43Delta x^3f'''(xi_2)$



            $a_2f(x+3Delta x) = a_2f(x) + a_23Delta xf(x) + a_2frac94Delta x^2f''(x) + a_2frac92Delta x^3f'''(xi_3) $



            this implies the augmented matrix:



            $left(beginarrayc
            1 & 1 & 1 & 0\
            1 & 2 & 3 & 1 \
            frac12 & 2 & frac94 & 0
            endarrayright)$



            do you see why?






            share|cite|improve this answer









            $endgroup$













              Your Answer








              StackExchange.ready(function()
              var channelOptions =
              tags: "".split(" "),
              id: "69"
              ;
              initTagRenderer("".split(" "), "".split(" "), channelOptions);

              StackExchange.using("externalEditor", function()
              // Have to fire editor after snippets, if snippets enabled
              if (StackExchange.settings.snippets.snippetsEnabled)
              StackExchange.using("snippets", function()
              createEditor();
              );

              else
              createEditor();

              );

              function createEditor()
              StackExchange.prepareEditor(
              heartbeatType: 'answer',
              autoActivateHeartbeat: false,
              convertImagesToLinks: true,
              noModals: true,
              showLowRepImageUploadWarning: true,
              reputationToPostImages: 10,
              bindNavPrevention: true,
              postfix: "",
              imageUploader:
              brandingHtml: "Powered by u003ca class="icon-imgur-white" href="https://imgur.com/"u003eu003c/au003e",
              contentPolicyHtml: "User contributions licensed under u003ca href="https://creativecommons.org/licenses/by-sa/3.0/"u003ecc by-sa 3.0 with attribution requiredu003c/au003e u003ca href="https://stackoverflow.com/legal/content-policy"u003e(content policy)u003c/au003e",
              allowUrls: true
              ,
              noCode: true, onDemand: true,
              discardSelector: ".discard-answer"
              ,immediatelyShowMarkdownHelp:true
              );



              );













              draft saved

              draft discarded


















              StackExchange.ready(
              function ()
              StackExchange.openid.initPostLogin('.new-post-login', 'https%3a%2f%2fmath.stackexchange.com%2fquestions%2f3171715%2fsecond-order-accurate-numerical-approximation-for-first-derivative%23new-answer', 'question_page');

              );

              Post as a guest















              Required, but never shown

























              2 Answers
              2






              active

              oldest

              votes








              2 Answers
              2






              active

              oldest

              votes









              active

              oldest

              votes






              active

              oldest

              votes









              2












              $begingroup$

              The formula you mention is a centered difference, but you are required to obtain a forward difference. You just need to use Taylor's formula and compute the coefficients that cancel out the lower order terms.



              $$
              F(x+Delta x)= F(x)+F'(x) Delta x + fracF''(x)2 (Delta x)^2 + O((Delta x)^3)
              $$



              $$
              F(x+ 2 Delta x)= F(x)+F'(x) 2 Delta x + fracF''(x)2 (2 Delta x)^2 + O((Delta x)^3)
              $$



              $$
              F(x+3Delta x)= F(x)+F'(x) 3Delta x + fracF''(x)2 (3Delta x)^2 + O((Delta x)^3)
              $$



              so you see that
              beginalign*
              F'(x)-frac1Delta x & (a_0 F(x+Delta x)+a_1 F(x+2Delta x) + a_2 F(x+3 Delta x))\
              =& frac1Delta xleft(F'(x) Delta x - a_0 (F(x)+F'(x) Delta x + fracF''(x)2 (Delta x)^2 + O((Delta x)^3))right.\
              & - a_1 (F(x)+F'(x) 2 Delta x + fracF''(x)2 (2 Delta x)^2 + O((Delta x)^3))\
              & left.- a_2 (F(x)+F'(x) 3Delta x + fracF''(x)2 (3Delta x)^2 + O((Delta x)^3))right)\
              =& frac1Delta xleft( -(a_0+a_1+a_2)F(x)+(1-a_0-2a_1-3a_2)F'(x)Delta xright.\
              & left.-(a_0+4a_1+9a_2)F''(x) (Delta x)^2/2 + O((Delta x)^3)right)
              endalign*



              To get the desired accuracy you just have to set $a_0, a_1, a_2$ such that



              $$
              a_0+a_1+a_2=0, quad a_0+2a_1+3a_2 = 1, quad a_0+4a_2+9a_2=0,
              $$



              that is
              $$
              a_0 = -frac 52, quad a_1 =4, quad a_2=-frac 32
              $$



              and the formula becomes
              $$
              F'(x) approx frac12Delta x left(-5 F(x+Delta x)+8F(x+2 Delta x) - 4F(x+3 Delta x) right)
              $$






              share|cite|improve this answer









              $endgroup$

















                2












                $begingroup$

                The formula you mention is a centered difference, but you are required to obtain a forward difference. You just need to use Taylor's formula and compute the coefficients that cancel out the lower order terms.



                $$
                F(x+Delta x)= F(x)+F'(x) Delta x + fracF''(x)2 (Delta x)^2 + O((Delta x)^3)
                $$



                $$
                F(x+ 2 Delta x)= F(x)+F'(x) 2 Delta x + fracF''(x)2 (2 Delta x)^2 + O((Delta x)^3)
                $$



                $$
                F(x+3Delta x)= F(x)+F'(x) 3Delta x + fracF''(x)2 (3Delta x)^2 + O((Delta x)^3)
                $$



                so you see that
                beginalign*
                F'(x)-frac1Delta x & (a_0 F(x+Delta x)+a_1 F(x+2Delta x) + a_2 F(x+3 Delta x))\
                =& frac1Delta xleft(F'(x) Delta x - a_0 (F(x)+F'(x) Delta x + fracF''(x)2 (Delta x)^2 + O((Delta x)^3))right.\
                & - a_1 (F(x)+F'(x) 2 Delta x + fracF''(x)2 (2 Delta x)^2 + O((Delta x)^3))\
                & left.- a_2 (F(x)+F'(x) 3Delta x + fracF''(x)2 (3Delta x)^2 + O((Delta x)^3))right)\
                =& frac1Delta xleft( -(a_0+a_1+a_2)F(x)+(1-a_0-2a_1-3a_2)F'(x)Delta xright.\
                & left.-(a_0+4a_1+9a_2)F''(x) (Delta x)^2/2 + O((Delta x)^3)right)
                endalign*



                To get the desired accuracy you just have to set $a_0, a_1, a_2$ such that



                $$
                a_0+a_1+a_2=0, quad a_0+2a_1+3a_2 = 1, quad a_0+4a_2+9a_2=0,
                $$



                that is
                $$
                a_0 = -frac 52, quad a_1 =4, quad a_2=-frac 32
                $$



                and the formula becomes
                $$
                F'(x) approx frac12Delta x left(-5 F(x+Delta x)+8F(x+2 Delta x) - 4F(x+3 Delta x) right)
                $$






                share|cite|improve this answer









                $endgroup$















                  2












                  2








                  2





                  $begingroup$

                  The formula you mention is a centered difference, but you are required to obtain a forward difference. You just need to use Taylor's formula and compute the coefficients that cancel out the lower order terms.



                  $$
                  F(x+Delta x)= F(x)+F'(x) Delta x + fracF''(x)2 (Delta x)^2 + O((Delta x)^3)
                  $$



                  $$
                  F(x+ 2 Delta x)= F(x)+F'(x) 2 Delta x + fracF''(x)2 (2 Delta x)^2 + O((Delta x)^3)
                  $$



                  $$
                  F(x+3Delta x)= F(x)+F'(x) 3Delta x + fracF''(x)2 (3Delta x)^2 + O((Delta x)^3)
                  $$



                  so you see that
                  beginalign*
                  F'(x)-frac1Delta x & (a_0 F(x+Delta x)+a_1 F(x+2Delta x) + a_2 F(x+3 Delta x))\
                  =& frac1Delta xleft(F'(x) Delta x - a_0 (F(x)+F'(x) Delta x + fracF''(x)2 (Delta x)^2 + O((Delta x)^3))right.\
                  & - a_1 (F(x)+F'(x) 2 Delta x + fracF''(x)2 (2 Delta x)^2 + O((Delta x)^3))\
                  & left.- a_2 (F(x)+F'(x) 3Delta x + fracF''(x)2 (3Delta x)^2 + O((Delta x)^3))right)\
                  =& frac1Delta xleft( -(a_0+a_1+a_2)F(x)+(1-a_0-2a_1-3a_2)F'(x)Delta xright.\
                  & left.-(a_0+4a_1+9a_2)F''(x) (Delta x)^2/2 + O((Delta x)^3)right)
                  endalign*



                  To get the desired accuracy you just have to set $a_0, a_1, a_2$ such that



                  $$
                  a_0+a_1+a_2=0, quad a_0+2a_1+3a_2 = 1, quad a_0+4a_2+9a_2=0,
                  $$



                  that is
                  $$
                  a_0 = -frac 52, quad a_1 =4, quad a_2=-frac 32
                  $$



                  and the formula becomes
                  $$
                  F'(x) approx frac12Delta x left(-5 F(x+Delta x)+8F(x+2 Delta x) - 4F(x+3 Delta x) right)
                  $$






                  share|cite|improve this answer









                  $endgroup$



                  The formula you mention is a centered difference, but you are required to obtain a forward difference. You just need to use Taylor's formula and compute the coefficients that cancel out the lower order terms.



                  $$
                  F(x+Delta x)= F(x)+F'(x) Delta x + fracF''(x)2 (Delta x)^2 + O((Delta x)^3)
                  $$



                  $$
                  F(x+ 2 Delta x)= F(x)+F'(x) 2 Delta x + fracF''(x)2 (2 Delta x)^2 + O((Delta x)^3)
                  $$



                  $$
                  F(x+3Delta x)= F(x)+F'(x) 3Delta x + fracF''(x)2 (3Delta x)^2 + O((Delta x)^3)
                  $$



                  so you see that
                  beginalign*
                  F'(x)-frac1Delta x & (a_0 F(x+Delta x)+a_1 F(x+2Delta x) + a_2 F(x+3 Delta x))\
                  =& frac1Delta xleft(F'(x) Delta x - a_0 (F(x)+F'(x) Delta x + fracF''(x)2 (Delta x)^2 + O((Delta x)^3))right.\
                  & - a_1 (F(x)+F'(x) 2 Delta x + fracF''(x)2 (2 Delta x)^2 + O((Delta x)^3))\
                  & left.- a_2 (F(x)+F'(x) 3Delta x + fracF''(x)2 (3Delta x)^2 + O((Delta x)^3))right)\
                  =& frac1Delta xleft( -(a_0+a_1+a_2)F(x)+(1-a_0-2a_1-3a_2)F'(x)Delta xright.\
                  & left.-(a_0+4a_1+9a_2)F''(x) (Delta x)^2/2 + O((Delta x)^3)right)
                  endalign*



                  To get the desired accuracy you just have to set $a_0, a_1, a_2$ such that



                  $$
                  a_0+a_1+a_2=0, quad a_0+2a_1+3a_2 = 1, quad a_0+4a_2+9a_2=0,
                  $$



                  that is
                  $$
                  a_0 = -frac 52, quad a_1 =4, quad a_2=-frac 32
                  $$



                  and the formula becomes
                  $$
                  F'(x) approx frac12Delta x left(-5 F(x+Delta x)+8F(x+2 Delta x) - 4F(x+3 Delta x) right)
                  $$







                  share|cite|improve this answer












                  share|cite|improve this answer



                  share|cite|improve this answer










                  answered Apr 4 at 8:04









                  PierreCarrePierreCarre

                  2,2781215




                  2,2781215





















                      1












                      $begingroup$

                      $a_0f(x+Delta x) = a_0f(x) + a_0Delta xf'(x) + a_0frac12Delta x^2f'(x) + a_0frac16Delta x^3f'''(xi_1)$



                      $a_1f(x+2Delta x) = a_1f(x) + a_12Delta xf'(x) +
                      a_12Delta x^2f''(x) + a_1frac43Delta x^3f'''(xi_2)$



                      $a_2f(x+3Delta x) = a_2f(x) + a_23Delta xf(x) + a_2frac94Delta x^2f''(x) + a_2frac92Delta x^3f'''(xi_3) $



                      this implies the augmented matrix:



                      $left(beginarrayc
                      1 & 1 & 1 & 0\
                      1 & 2 & 3 & 1 \
                      frac12 & 2 & frac94 & 0
                      endarrayright)$



                      do you see why?






                      share|cite|improve this answer









                      $endgroup$

















                        1












                        $begingroup$

                        $a_0f(x+Delta x) = a_0f(x) + a_0Delta xf'(x) + a_0frac12Delta x^2f'(x) + a_0frac16Delta x^3f'''(xi_1)$



                        $a_1f(x+2Delta x) = a_1f(x) + a_12Delta xf'(x) +
                        a_12Delta x^2f''(x) + a_1frac43Delta x^3f'''(xi_2)$



                        $a_2f(x+3Delta x) = a_2f(x) + a_23Delta xf(x) + a_2frac94Delta x^2f''(x) + a_2frac92Delta x^3f'''(xi_3) $



                        this implies the augmented matrix:



                        $left(beginarrayc
                        1 & 1 & 1 & 0\
                        1 & 2 & 3 & 1 \
                        frac12 & 2 & frac94 & 0
                        endarrayright)$



                        do you see why?






                        share|cite|improve this answer









                        $endgroup$















                          1












                          1








                          1





                          $begingroup$

                          $a_0f(x+Delta x) = a_0f(x) + a_0Delta xf'(x) + a_0frac12Delta x^2f'(x) + a_0frac16Delta x^3f'''(xi_1)$



                          $a_1f(x+2Delta x) = a_1f(x) + a_12Delta xf'(x) +
                          a_12Delta x^2f''(x) + a_1frac43Delta x^3f'''(xi_2)$



                          $a_2f(x+3Delta x) = a_2f(x) + a_23Delta xf(x) + a_2frac94Delta x^2f''(x) + a_2frac92Delta x^3f'''(xi_3) $



                          this implies the augmented matrix:



                          $left(beginarrayc
                          1 & 1 & 1 & 0\
                          1 & 2 & 3 & 1 \
                          frac12 & 2 & frac94 & 0
                          endarrayright)$



                          do you see why?






                          share|cite|improve this answer









                          $endgroup$



                          $a_0f(x+Delta x) = a_0f(x) + a_0Delta xf'(x) + a_0frac12Delta x^2f'(x) + a_0frac16Delta x^3f'''(xi_1)$



                          $a_1f(x+2Delta x) = a_1f(x) + a_12Delta xf'(x) +
                          a_12Delta x^2f''(x) + a_1frac43Delta x^3f'''(xi_2)$



                          $a_2f(x+3Delta x) = a_2f(x) + a_23Delta xf(x) + a_2frac94Delta x^2f''(x) + a_2frac92Delta x^3f'''(xi_3) $



                          this implies the augmented matrix:



                          $left(beginarrayc
                          1 & 1 & 1 & 0\
                          1 & 2 & 3 & 1 \
                          frac12 & 2 & frac94 & 0
                          endarrayright)$



                          do you see why?







                          share|cite|improve this answer












                          share|cite|improve this answer



                          share|cite|improve this answer










                          answered Apr 4 at 5:36









                          GeauxMathGeauxMath

                          1114




                          1114



























                              draft saved

                              draft discarded
















































                              Thanks for contributing an answer to Mathematics Stack Exchange!


                              • Please be sure to answer the question. Provide details and share your research!

                              But avoid


                              • Asking for help, clarification, or responding to other answers.

                              • Making statements based on opinion; back them up with references or personal experience.

                              Use MathJax to format equations. MathJax reference.


                              To learn more, see our tips on writing great answers.




                              draft saved


                              draft discarded














                              StackExchange.ready(
                              function ()
                              StackExchange.openid.initPostLogin('.new-post-login', 'https%3a%2f%2fmath.stackexchange.com%2fquestions%2f3171715%2fsecond-order-accurate-numerical-approximation-for-first-derivative%23new-answer', 'question_page');

                              );

                              Post as a guest















                              Required, but never shown





















































                              Required, but never shown














                              Required, but never shown












                              Required, but never shown







                              Required, but never shown

































                              Required, but never shown














                              Required, but never shown












                              Required, but never shown







                              Required, but never shown







                              Popular posts from this blog

                              Triangular numbers and gcdProving sum of a set is $0 pmod n$ if $n$ is odd, or $fracn2 pmod n$ if $n$ is even?Is greatest common divisor of two numbers really their smallest linear combination?GCD, LCM RelationshipProve a set of nonnegative integers with greatest common divisor 1 and closed under addition has all but finite many nonnegative integers.all pairs of a and b in an equation containing gcdTriangular Numbers Modulo $k$ - Hit All Values?Understanding the Existence and Uniqueness of the GCDGCD and LCM with logical symbolsThe greatest common divisor of two positive integers less than 100 is equal to 3. Their least common multiple is twelve times one of the integers.Suppose that for all integers $x$, $x|a$ and $x|b$ if and only if $x|c$. Then $c = gcd(a,b)$Which is the gcd of 2 numbers which are multiplied and the result is 600000?

                              Ingelân Ynhâld Etymology | Geografy | Skiednis | Polityk en bestjoer | Ekonomy | Demografy | Kultuer | Klimaat | Sjoch ek | Keppelings om utens | Boarnen, noaten en referinsjes Navigaasjemenuwww.gov.ukOffisjele webside fan it regear fan it Feriene KeninkrykOffisjele webside fan it Britske FerkearsburoNederlânsktalige ynformaasje fan it Britske FerkearsburoOffisjele webside fan English Heritage, de organisaasje dy't him ynset foar it behâld fan it Ingelske kultuergoedYnwennertallen fan alle Britske stêden út 'e folkstelling fan 2011Notes en References, op dizze sideEngland

                              Հադիս Բովանդակություն Անվանում և նշանակություն | Դասակարգում | Աղբյուրներ | Նավարկման ցանկ